2
$\begingroup$

Let's consider the following PDE in $\mathbb R^d$ : $$\frac{\partial^d u}{\partial x_1...\partial x_d}=f$$ where $f$ is a tempered distribution with support in $\mathbb R^d_+$. There is a result by Hörmander that states that there exists at least one tempered distribution solution to this PDE (this has to do with the division of tempered distributions by polynomials). I want to know if (because of the particularly simple form of this particular equation) all solutions to this PDE are in fact tempered?

$\endgroup$

1 Answer 1

4
$\begingroup$

This is false. For example for $d=2$ you have solutions of the form $F(x,y)+g(x)+h(y)$ where $F$ is the standard tempered solution obtained by integration and $g$ and $h$ are ANY distributions, hence not necessarily tempered.

$\endgroup$
8
  • $\begingroup$ Thank you! What if I add boundary conditions, for example impose the solution $u$ to have support in $\mathbb R_+^d$? $\endgroup$
    – Thomas
    Apr 28, 2014 at 11:15
  • $\begingroup$ That doesn't change things. Consider the special case with $f=0$ which is particularly transparent $\endgroup$
    – janacek
    Apr 28, 2014 at 11:46
  • $\begingroup$ It seems to me that it does. In the case $f=0$, all distributions of the form $u=g\otimes 1+ 1\otimes h $ are solutions where $g$ and $h$ are any distributions. But then the support condition imposes that $g$ and $h$ are in fact $0$, so all these solutions are tempered. $\endgroup$
    – Thomas
    Apr 28, 2014 at 12:04
  • $\begingroup$ just take $u=H(x)\exp(x)$ and so on. Or are we talking at cross purposes? $\endgroup$
    – janacek
    Apr 28, 2014 at 12:48
  • $\begingroup$ This particular $u$ does not have support in $\mathbb R_+^2$: if I choose $\varphi, \psi \in \mathcal D(\mathbb R)$ such that $\text{supp} \, \varphi \subset \mathbb R_+$ and $\text{supp} \, \psi \subset \mathbb R_-$ then $\text{supp} \, \varphi \otimes \psi \cap \mathbb R_+^2=\emptyset$ and $\langle u , \varphi \otimes \psi \rangle \neq 0$. $\endgroup$
    – Thomas
    Apr 28, 2014 at 13:31

Your Answer

By clicking “Post Your Answer”, you agree to our terms of service and acknowledge you have read our privacy policy.

Not the answer you're looking for? Browse other questions tagged or ask your own question.